LSAT and Law School Admissions Forum

Get expert LSAT preparation and law school admissions advice from PowerScore Test Preparation.

 Administrator
PowerScore Staff
  • PowerScore Staff
  • Posts: 8919
  • Joined: Feb 02, 2011
|
#41424
Please post your questions below!
 Bruin96
  • Posts: 33
  • Joined: Sep 04, 2019
|
#71738
Can someone please explain why answer choice C is correct?
 Rachael Wilkenfeld
PowerScore Staff
  • PowerScore Staff
  • Posts: 1358
  • Joined: Dec 15, 2011
|
#71765
Hi Bruin96,

This argument is causal. The store owner tells his friend he heard that bright lights CAUSES a reduction in vandalism. Three month later, there's been a reduction in vandalism. So the friend concludes that the bright lights were installed. Our question stem is a weaken except question, meaning that four of our answer choices will weaken the argument, and one will not weaken it.

Causal reasoning on the LSAT is pretty easy to weaken. Any time we see the cause without the effect, the effect without the cause, the cause and effect reversed, or an alternate cause, we weaken that causal relationship. Here, we know we have the effect---the stimulus tells us there's been a reduction in vandalism. So we expect our answer choices to be alternate causes or otherwise attack the cause (bright lights).

Answer choice (A): This weakens the conclusion by providing an alternate cause--an increased police presence in the area.

Answer choice (B): This weakens the conclusion by stating the cause could not yet have occured. If the bright lights were the cause, they wouldn't have been installed yet. And, since the cause must occur before the effect can occur, this would weaken the argument.

Answer choice (C): This is the correct answer choice. This answer choice is consistent with the cause of the reduction being bright lights. The stores on the perimeter would have benefited from the store owner's lights, while the stores one block away would not have. As this does not weaken the causal relationship, but instead is consistent with it, it is our correct answer.

Answer choice (D): This weakens the conclusion by giving a reason that the bright lights would not have been installed.

Answer choice (E): Much like answer choice (A), this answer choice gives us an alternate cause for the reduction in vandalism.

Hope that helps!
Rachael

Get the most out of your LSAT Prep Plus subscription.

Analyze and track your performance with our Testing and Analytics Package.